Ch. 48 Diabetes Mellitus

Pataasin ang iyong marka sa homework at exams ngayon gamit ang Quizwiz!

The nurse has been teaching a patient with type 2 diabetes about managing blood glucose levels and taking glipizide (Glucotrol). Which patient statement indicates a need for additional teaching? a. "If I overeat at a meal, I will still take the usual dose of medication." b. "Other medications besides the Glucotrol may affect my blood sugar." c. "When I am ill, I may have to take insulin to control my blood sugar." d. "My diabetes won't cause complications because I don't need insulin."

D

1. Which statement by a nurse to a patient newly diagnosed with type 2 diabetes is correct? a. Insulin is not used to control blood glucose in patients with type 2 diabetes. b. Complications of type 2 diabetes are less serious than those of type 1 diabetes. c. Changes in diet and exercise may control blood glucose levels in type 2 diabetes. d. Type 2 diabetes is usually diagnosed when the patient is admitted with a hyperglycemic coma.

ANS: C For some patients with type 2 diabetes, changes in lifestyle are sufficient to achieve blood glucose control. Insulin is frequently used for type 2 diabetes, complications are equally severe as for type 1 diabetes, and type 2 diabetes is usually diagnosed with routine laboratory testing or after a patient develops complications such as frequent yeast infections. DIF: Cognitive Level: Understand (comprehension) REF: 1134 TOP: Nursing Process: Implementation MSC: NCLEX: Physiological Integrity

39. When a patient with type 2 diabetes is admitted for a cholecystectomy, which nursing action can the nurse delegate to a licensed practical/vocational nurse (LPN/LVN)? a. Communicate the blood glucose level and insulin dose to the circulating nurse in surgery. b. Discuss the reason for the use of insulin therapy during the immediate postoperative period. c. Administer the prescribed lispro (Humalog) insulin before transporting the patient to surgery. d. Plan strategies to minimize the risk for hypoglycemia or hyperglycemia during the postoperative period.

ANS: C LPN/LVN education and scope of practice includes administration of insulin. Communication about patient status with other departments, planning, and patient teaching are skills that require RN education and scope of practice. DIF: Cognitive Level: Apply (application) REF: 1152 OBJ: Special Questions: Delegation TOP: Nursing Process: Planning MSC: NCLEX: Safe and Effective Care Environment

29. A patient who has type 2 diabetes is being prepared for an elective coronary angiogram. Which information would the nurse anticipate might lead to rescheduling the test? a. The patient's most recent A1C was 6.5%. b. The patient's blood glucose is 128 mg/dL. c. The patient took the prescribed metformin today. d. The patient took the prescribed captopril this morning.

ANS: C To avoid lactic acidosis, metformin should be discontinued a day or 2 before the coronary angiogram and should not be used for 48 hours after IV contrast media are administered. The other patient data will also be reported but do not indicate any need to reschedule the procedure. DIF: Cognitive Level: Apply (application) TOP: Nursing Process: AssessmREF: 1130 MSC: NCLEX: Physiological Integrity

31. A patient with diabetes rides a bicycle to and from work every day. Which site should the nurse teach the patient to use to administer the morning insulin? a. thigh. b. buttock. c. abdomen. d. upper arm.

ANS: C Patients should be taught not to administer insulin into a site that will be exercised because exercise will increase the rate of absorption. The thigh, buttock, and arm are all exercised by riding a bicycle. DIF: Cognitive Level: Apply (application) TOP: Nursing Process: Implementation REF: 1128 MSC: NCLEX: Physiological Integrit

4. The nurse is assessing a 22-yr-old patient experiencing the onset of symptoms of type 1 diabetes. To which question would the nurse anticipate a positive response? a. "Are you anorexic?" b. "Is your urine dark colored?" c. "Have you lost weight lately?" d. "Do you crave sugary drinks?"

ANS: C Weight loss occurs because the body is no longer able to absorb glucose and starts to break down protein and fat for energy. The patient is thirsty but does not necessarily crave sugar-containing fluids. Increased appetite is a classic symptom of type 1 diabetes. With the classic symptom of polyuria, urine will be very dilute. DIF: Cognitive Level: Apply (application) TOP: Nursing Process: Assessment REF: 1121 MSC: NCLEX: Physiological Integrity

32. The nurse is interviewing a new patient with diabetes who takes rosiglitazone (Avandia). Which information would the nurse anticipate resulting in the health care provider discontinuing the medication? a. The patient's blood pressure is 154/92. b. The patient's blood glucose is 86 mg/dL. c. The patient reports a history of emphysema. d. The patient has chest pressure when walking.

ANS: D Rosiglitazone can cause myocardial ischemia. The nurse should immediately notify the health care provider and expect orders to discontinue the medication. A blood glucose level of 86 mg/dL indicates a positive effect from the medication. Hypertension and a history of emphysema do not contraindicate this medication. DIF: Cognitive Level: Apply (application) REF: 1130 TOP: Nursing Process: Assessment MSC: NCLEX: Physiological Integrity

A patient who has type 2 diabetes is being prepared for an elective coronary angiogram. Which information would the nurse anticipate might lead to rescheduling the test?

The pt took metformin today

25. Which question during the assessment of a patient who has diabetes will help the nurse identify autonomic neuropathy? a. "Do you feel bloated after eating?" b. "Have you seen any skin changes?" c. "Do you need to increase your insulin dosage when you are stressed?" d. "Have you noticed any painful new ulcerations or sores on your feet?"

a. "Do you feel bloated after eating?" - Autonomic neuropathy can cause delayed gastric emptying, which results in a bloated feeling for the patient. The other questions are also appropriate to ask but would not help in identifying autonomic neuropathy

16. The nurse has been teaching a patient with type 2 diabetes about managing blood glucose levels and taking glipizide (Glucotrol). Which patient statement indicates a need for additional teaching? a. "If I overeat at a meal, I will still take the usual dose of med.." b. "Other med. besides the Glucotrol may affect my blood sugar." c. "When I am ill, I may have to take insulin to control my blood sugar." d. "My diabetes won't cause complications because I don't need insulin."

ANS: D The patient should understand that type 2 diabetes places the patient at risk for many complications and that good glucose control is as important when taking oral agents as when using insulin. The other statements are accurate and indicate good understanding of the use of glipizide. DIF: Cognitive Level: Apply (application) TOP: Nursing Process: Evaluation REF: 1130 MSC: NCLEX: Physiological Integrity

The nurse is assigned to the care of a patient diagnosed with type 2 diabetes. In formulating a teaching plan that encourages the patient to actively participate in management of the diabetes, what should be the nurse's initial intervention? Assess patient's perception of what it means to have diabetes. Ask the patient to write down current knowledge about diabetes. Set goals for the patient to actively participate in managing his diabetes. Assume responsibility for all of the patient's care to decrease stress level.

Assess patient's perception of what it means to have diabetes.

A few weeks after an 82-yr-old patient with a new diagnosis of type 2 diabetes has been placed on metformin (Glucophage) therapy, the home health nurse makes a visit. Which finding should the nurse promptly discuss with the health care provider? a. Hemoglobin A1C level is 7.9%. b. Glomerular filtration rate is decreased. c. Last eye examination was 18 months ago. d. Patient has questions about the prescribed diet.

B

After change-of-shift report, which patient should the nurse assess first? a. A 19-yr-old patient with type 1 diabetes who has a hemoglobin A1C of 12% b. A 23-yr-old patient with type 1 diabetes who has a blood glucose of 40 mg/dL c. A 50-yr-old patient who uses exenatide (Byetta) and is reporting acute abdominal pain d. A 40-yr-old patient who is pregnant and whose oral glucose tolerance test is 202 mg/dL

B

Which action by the patient who is self-monitoring blood glucose indicates a need for additional teaching? a. Washes the puncture site using warm water and soap. b. Chooses a puncture site in the center of the finger pad. c. Hangs the arm down for a minute before puncturing the site. d. Says the result of 120 mg indicates good blood sugar control.

B

Which action should the nurse take after a patient treated with intramuscular glucagon for hypoglycemia regains consciousness? a. Assess the patient for symptoms of hyperglycemia. b. Give the patient a snack of peanut butter and crackers. c. Have the patient drink a glass of orange juice or nonfat milk. d. Administer a continuous infusion of 5% dextrose for 24 hours.

B

Which patient action indicates accurate understanding of the nurse's teaching about administration of aspart (NovoLog) insulin? a. The patient avoids injecting the insulin into the upper abdominal area. b. The patient cleans the skin with soap and water before insulin administration. c. The patient stores the insulin in the freezer after administering the prescribed dose. d. The patient pushes the plunger down while removing the syringe from the injection site.

B Cleaning the skin with soap and water is acceptable. Insulin should not be frozen. The patient should leave the syringe in place for about 5 seconds after injection to be sure that all the insulin has been injected. The upper abdominal area is one of the preferred areas for insulin injection.

An unresponsive patient who has type 2 diabetes is brought to the emergency department and diagnosed with hyperosmolar hyperglycemia syndrome (HHS). What should the nurse anticipate doing? a. Giving 50% dextrose b. Inserting an IV catheter c. Initiating O2 by nasal cannula d. Administering glargine (Lantus) insulin

B HHS is initially treated with large volumes of IV fluids to correct hypovolemia. Regular insulin is administered, not a long-acting insulin. There is no indication that the patient requires O2. Dextrose solutions will increase the patient's blood glucose and would be contraindicated.

Which statement by the patient who has newly diagnosed type 1 diabetes indicates a need for additional instruction from the nurse? a. "I will need a bedtime snack because I take an evening dose of NPH insulin." b. "I can choose any foods, as long as I use enough insulin to cover the calories." c. "I can have an occasional beverage with alcohol if I include it in my meal plan." d. "I will eat something at meal times to prevent hypoglycemia, even if I am not hungry."

B Most patients with type 1 diabetes need to plan diet choices very carefully. Patients who are using intensified insulin therapy have considerable flexibility in diet choices but still should restrict dietary intake of items such as fat, protein, and alcohol. The other patient statements are correct and indicate good understanding of the diet instruction.

The nurse is taking a health history from a 29-yr-old patient at the first prenatal visit. The patient reports that she has no personal history of diabetes, but her mother has diabetes. Which action will the nurse plan to take? a. Teach the patient about administering regular insulin. b. Schedule the patient for a fasting blood glucose level. c. Teach about an increased risk for fetal problems with gestational diabetes. d. Schedule an oral glucose tolerance test for the twenty-fourth week of pregnancy.

B Patients at high risk for gestational diabetes should be screened for diabetes on the initial prenatal visit. An oral glucose tolerance test may also be used to check for diabetes, but it would be done before the twenty-fourth week. Teaching plans would depend on the outcome of a fasting blood glucose test and other tests.

which statement to the nurse indicates a need for additional instruction in administering insulin? a. "I can buy the 0.5-mL syringes because the line markings are easier to see." b. "I need to rotate injection sites among my arms, legs, and abdomen each day." c. "I do not need to aspirate the plunger to check for blood before injecting insulin." d. "I should draw up the regular insulin first, after injecting air into the NPH bottle."

B Rotating sites is no longer recommended because there is more consistent insulin absorption when the same site is used consistently. The other patient statements are accurate and indicate that no additional instruction is needed.

Which laboratory value reported by the unlicensed assistive personnel (UAP) indicates an urgent need for the nurse to assess the patient? a. Bedtime glucose of 140 mg/dL b. Noon blood glucose of 52 mg/dL c. Fasting blood glucose of 130 mg/dL d. 2-hr postprandial glucose of 220 mg/dL

B The nurse should assess the patient with a blood glucose level of 52 mg/dL for symptoms of hypoglycemia and give the patient a carbohydrate-containing beverage such as orange juice. The other values are within an acceptable range or not immediately dangerous for a patient with diabetes.

Which information will the nurse include when teaching a patient who has type 2 diabetes about glyburide? a. Glyburide decreases glucagon secretion from the pancreas. b. Glyburide stimulates insulin production and release from the pancreas. c. Glyburide should be taken even if the morning blood glucose level is low. d. Glyburide should not be used for 48 hours after receiving IV contrast media.

B The sulfonylureas stimulate the production and release of insulin from the pancreas. If the glucose level is low, the patient should contact the health care provider before taking glyburide because hypoglycemia can occur with this class of medication. Metformin should be held for 48 hours after administration of IV contrast media, but this is not necessary for glyburide. Glyburide does not affect glucagon secretion.

A patient receives aspart (NovoLog) insulin at 8:00 AM. At which time would the nurse anticipate the highest risk for hypoglycemia? a. 10:00 AM b. 12:00 AM c. 2:00 PM d. 4:00 PM

A The rapid-acting insulins peak in 1 to 3 hours. The patient is not at a high risk for hypoglycemia at the other listed times, although hypoglycemia may occur.

1. To monitor for complications in a patient with type 2 diabetes, which tests will the nurse in the diabetic clinic schedule at least annually (select all that apply)? a. Chest x-ray b. Blood pressure c. Serum creatinine d. Urine for microalbuminuria e. Complete blood count (CBC) f. Monofilament testing of the foot

B, C, D, F

The health care provider suspects the Somogyi effect in a 50-yr-old patient whose 6:00 AMblood glucose is 230 mg/dL. Which action will the nurse teach the patient to take? a. Avoid snacking right before bedtime. b. Increase the rapid-acting insulin dose. c. Check the blood glucose during the night. d. Administer a larger dose of long-acting insulin.

C

A 26-yr-old female who has type 1 diabetes develops a sore throat and runny nose after caring for her sick toddler. The patient calls the clinic for advice about her symptoms and reports a blood glucose level of 210 mg/dL despite taking her usual glargine (Lantus) and lispro (Humalog) insulin. What should the nurse advise the patient to do? a. Use only the lispro insulin until the symptoms are resolved. b. Limit intake of calories until the glucose is less than 120 mg/dL. c. Monitor blood glucose every 4 hours and contact the clinic if it rises. d. Decrease carbohydrates until glycosylated hemoglobin is less than 7%.

C

An active 32-yr-old male who has type 1 diabetes is being seen in the endocrine clinic. Which finding indicates a need for the nurse to discuss a possible a change in therapy with the health care provider? a. Hemoglobin A1C level of 6.2% b. Heart rate at rest of 58 beats/min c. Blood pressure of 140/88 mmHg d. High-density lipoprotein (HDL) level of 65 mg/dL

C

Which patient statement indicates that the nurse's teaching about exenatide (Byetta) has been effective? a. "I may feel hungrier than usual when I take this medicine." b. "I will not need to worry about hypoglycemia with the Byetta." c. "I should take my daily aspirin at least an hour before the Byetta." d. "I will take the pill at the same time I eat breakfast in the morning."

C Because exenatide slows gastric emptying, oral medications should be taken at least 1 hour before the exenatide to avoid slowing absorption. Exenatide is injected and increases feelings of satiety. Hypoglycemia can occur with this medication.

A 30-yr-old patient has a new diagnosis of type 2 diabetes. When should the nurse recommend the patient schedule a dilated eye examination? a. Every 2 years b. Every 6 months c. As soon as available d. At the age of 39 years

C Because many patients have some diabetic retinopathy when they are first diagnosed with type 2 diabetes, a dilated eye examination is recommended at the time of diagnosis and annually thereafter.

A patient with type 2 diabetes has a urinary tract infection (UTI), is difficult to arouse, and has a blood glucose of 642 mg/dL. When the nurse assesses the urine, there are no ketones present. What nursing action is appropriate at this time? Routine insulin therapy and exercise Administer a different antibiotic for the UTI. Cardiac monitoring to detect potassium changes Administer IV fluids rapidly to correct dehydration.

Cardiac monitoring to detect potassium changes

The nurse is assisting a patient with newly diagnosed type 2 diabetes to learn dietary planning as part of the initial management of diabetes. The nurse would encourage the patient to limit intake of which foods to help reduce the percent of fat in the diet? Cheese Broccoli Chicken Oranges

Cheese

which information will the nurse include in teaching a female patient who has peripheral arterial disease, type 2 diabetes, and sensory neuropathy of the feet and legs?

Choose flat-soled leather shoes

The nurse has been teaching a patient with diabetes mellitus how to perform self-monitoring of blood glucose (SMBG). During evaluation of the patient's technique, the nurse identifies a need for additional teaching when the patient does what? Chooses a puncture site in the center of the finger pad Washes hands with soap and water to cleanse the site to be used Warms the finger before puncturing the finger to obtain a drop of blood Tells the nurse that the result of 110 mg/dL indicates good control of diabetes

Chooses a puncture site in the center of the finger pad

10. Which statement by the patient indicates a need for additional instruction in administering insulin? a. "I need to rotate injection sites among my arms, legs, and abdomen each day." b. "I can buy the 0.5-mL syringes because the line markings will be easier to see." c. "I do not need to aspirate the plunger to check for blood before injecting insulin." d. "I should draw up the regular insulin first, after injecting air into the NPH bottle."

ANS: A Rotating sites is no longer recommended because there is more consistent insulin absorption when the same site is used consistently. The other patient statements are accurate and indicate that no additional instruction is needed. DIF: Cognitive Level: Apply (application) TOP: Nursing Process: Evaluation REF: 1128 MSC: NCLEX: Health Promotion and Maintenance

After change-of-shift report, which patient will the nurse assess first? a. A 19-yr-old patient with type 1 diabetes who was admitted with dawn phenomenon b. A 35-yr-old patient with type 1 diabetes whose most recent blood glucose reading was 230 mg/dL c. A 68-yr-old patient with type 2 diabetes who has severe peripheral neuropathy and reports burning foot pain d. A 60-yr-old patient with hyperosmolar hyperglycemic syndrome who has poor skin turgor and dry oral mucosa

D

Which finding indicates a need to contact the health care provider before the nurse administers metformin (Glucophage)? a. The patient's blood glucose level is 174 mg/dL. b. The patient is scheduled for a chest x-ray in an hour. c. The patient has gained 2 lb (0.9 kg) in the past 24 hours. d. The patient's estimated glomerular filtration rate is 42 mL/min.

D The glomerular filtration rate indicates possible renal impairment, and metformin should not be used in patients with significant renal impairment. The other findings are not contraindications to the use of metformin.

40. An active 32-yr-old male who has type 1 diabetes is being seen in the endocrine clinic. Which finding indicates a need for the nurse to discuss a possible a change in therapy with the health care provider? a. Hemoglobin A1C level of 6.2% b. Blood pressure of 140/88 mmHg c. Heart rate at rest of 58 beats/minute d. High density lipoprotein (HDL) level of 65 mg/dL

b. Blood pressure of 140/88 mmHg - To decrease the incidence of macrovascular and microvascular problems in patients with diabetes, the goal blood pressure is usually 130/80 mm Hg. An A1C less than 6.5%, a low resting heart rate (consistent with regular aerobic exercise in a young adult), and an HDL level of 65 mg/dL all indicate that the patient's diabetes and risk factors for vascular disease are well controlled.

A patient is newly diagnosed with type 1 diabetes and reports a headache, changes in vision, and being anxious but does not have a portable blood glucose monitor present. Which action should the nurse advise her to take? Eat a piece of pizza. Drink some diet pop. Eat 15 g of simple carbohydrates. Take an extra dose of rapid-acting insulin.

Eat 15 g of simple carbohydrates.

The nurse is evaluating a patient diagnosed with type 2 diabetes mellitus. Which symptom reported by the patient correlates with the diagnosis? Excessive thirst Gradual weight gain Overwhelming fatigue Recurrent blurred vision

Excessive thirst

The nurse is reviewing laboratory results for a patient with a 15-year history of type 2 diabetes. Which result reflects the expected pattern accompanying macrovascular disease as a complication of diabetes? Increased triglyceride levels Increased high-density lipoproteins (HDL) Decreased low-density lipoproteins (LDL) Decreased very-low-density lipoproteins (VLDL)

Increased triglyceride levels

A patient, admitted with diabetes mellitus, has a glucose level of 380 mg/dL and a moderate level of ketones in the urine. As the nurse assesses for signs of ketoacidosis, which respiratory pattern would the nurse expect to find? Central apnea Hypoventilation Kussmaul respirations Cheyne-Stokes respirations

Kussmaul respirations

A patient with diabetes mellitus who has multiple infections every year needs a mitral valve replacement. What is the most important preoperative teaching the nurse should provide to prevent a cardiac infection postoperatively? Avoid sick people and wash hands. Obtain comprehensive dental care. Maintain hemoglobin A1C below 7%. Coughing and deep breathing with splinting

Obtain comprehensive dental care.

The newly diagnosed patient with type 2 diabetes has been prescribed metformin. What should the nurse teach the patient to best explain how this medication works? Increases insulin production from the pancreas Slows the absorption of carbohydrate in the small intestine Reduces glucose production by the liver and enhances insulin sensitivity Increases insulin release from the pancreas, inhibits glucagon secretion, and decreases gastric emptying

Reduces glucose production by the liver and enhances insulin sensitivity

The nurse is teaching a patient who has diabetes about vascular complications of diabetes. What information is appropriate for the nurse to include? Macroangiopathy does not occur in type 1 diabetes but rather in type 2 diabetics who have severe disease. Microangiopathy is specific to diabetes and most commonly affects the capillary membranes of the eyes, kidneys, and skin. Renal damage resulting from changes in large- and medium-sized blood vessels can be prevented by careful glucose control. Macroangiopathy causes slowed gastric emptying and the sexual impotency experienced by a majority of patients with diabetes.

Microangiopathy is specific to diabetes and most commonly affects the capillary membranes of the eyes, kidneys, and skin.

A patient with diabetes mellitus is scheduled for a fasting blood glucose level at 8:00 AM. The nurse instructs the patient to only drink water after what time? 6:00 PM on the evening before the test Midnight before the test 4:00 AM on the day of the test 7:00 AM on the day of the test

Midnight before the test

What is the priority action for the nurse to take if the patient with type 2 diabetes complains of blurred vision and irritability? a. Call the physician. b. Administer insulin as ordered. c. Check the patient's blood glucose level. d. Assess for other neurologic symptoms.

c. Check the patient's blood glucose level.

4. The nurse is assessing a 22-yr-old patient experiencing the onset of symptoms of type 1 diabetes. To which question would the nurse anticipate a positive response? a. "Are you anorexic?" b. "Is your urine dark colored?" c. "Have you lost weight lately?" d. "Do you crave sugary drinks?"

c. "Have you lost weight lately?" - Weight loss occurs because the body is no longer able to absorb glucose and starts to break down protein and fat for energy. The patient is thirsty but does not necessarily crave sugar-containing fluids. Increased appetite is a classic symptom of type 1 diabetes. With the classic symptom of polyuria, urine will be very dilute.

The nurse is teaching a patient with type 2 diabetes mellitus how to prevent diabetic nephropathy. Which statement made by the patient indicates that teaching has been successful? "Smokeless tobacco products decrease the risk of kidney damage." "I can help control my blood pressure by avoiding foods high in salt." "I should have yearly dilated eye examinations by an ophthalmologist." "I will avoid hypoglycemia by keeping my blood sugar above 180 mg/dL."

"I can help control my blood pressure by avoiding foods high in salt."

The nurse instructs a patient with diabetes mellitus about a healthy eating plan. Which statement made by the patient indicates that teaching was successful? "I plan to lose 25 lb this year by following a high-protein diet." "I may have a hypoglycemic reaction if I drink alcohol on an empty stomach." "I should include more fiber in my diet than a person who does not have diabetes." "If I use an insulin pump, I will not need to limit the amount of saturated fat in my diet."

"I may have a hypoglycemic reaction if I drink alcohol on an empty stomach."

The nurse is interviewing a new patient with diabetes who takes rosiglitazone (Avandia). Which information would the nurse anticipate resulting in the health care provider discontinuing the medication?

The patient has chest posture when walking

5. A patient with type 2 diabetes is scheduled for a follow-up visit in the clinic several months from now. Which test will the nurse schedule to evaluate the effectiveness of treatment for the patient? a. Fasting blood glucose b. Oral glucose tolerance c Glycosylated hemoglobin d. Urine dipstick for glucose

_ANS: C The glycosylated hemoglobin (A1C) test shows the overall control of glucose over 90 to 120 days. A fasting blood level indicates only the glucose level at one time. Urine glucose testing is not an accurate reflection of blood glucose level and does not reflect the glucose over a prolonged time. Oral glucose tolerance testing is done to diagnose diabetes but is not used for monitoring glucose control after diabetes has been diagnosed. DIF: Cognitive Level: Apply (application) TOP: Nursing Process: Planning REF: 1124 MSC: NCLEX: Physiological Integrity

14. A patient with diabetes is starting on intensive insulin therapy. Which type of insulin will the nurse discuss using for mealtime coverage? a. Lispro (Humalog) b. Glargine (Lantus) c. Detemir (Levemir) d. NPH (Humulin N)

a. Lispro (Humalog) - Rapid- or short-acting insulin is used for mealtime coverage for patients receiving intensive insulin therapy. NPH, glargine, or detemir will be used as the basal insulin.

10. Which statement by the patient indicates a need for additional instruction in administering insulin? a. "I need to rotate injection sites among my arms, legs, and abdomen each day." b. "I can buy the 0.5-mL syringes because the line markings will be easier to see." c. "I do not need to aspirate the plunger to check for blood before injecting insulin." d. "I should draw up the regular insulin first, after injecting air into the NPH bottle."

a. "I need to rotate injection sites among my arms, legs, and abdomen each day." - Rotating sites is no longer recommended because there is more consistent insulin absorption when the same site is used consistently. The other patient statements are accurate and indicate that no additional instruction is needed.

12. A patient receives aspart (NovoLog) insulin at 8:00 AM. At which time would the nurse anticipate the highest risk for hypoglycemia? a. 10:00 AM b. 12:00 AM c. 2:00 PM d. 4:0 PM

a. 10:00 AM - The rapid-acting insulins peak in 1 to 3 hours. The patient is not at a high risk for hypoglycemia at the other listed times, although hypoglycemia may occur.

Analyze the following diagnostic findings for your patient with type 2 diabetes. Which result will need further assessment? a. A1C 9% b. BP 126/80 mm Hg c. FBG 130 mg/dL (7.2 mmol/L) d. LDL cholesterol 100 mg/dL (2.6 mmol/L)

a. A1C 9%

26. Which information will the nurse include in teaching a female patient who has peripheral arterial disease, type 2 diabetes, and sensory neuropathy of the feet and legs? a. Choose flat-soled leather shoes. b. Set heating pads on a low temperature. c. Use callus remover for corns or calluses. d. Soak feet in warm water for an hour each day.

a. Choose flat-soled leather shoes. - The patient is taught to avoid high heels and that leather shoes are preferred. The feet should be washed, but not soaked, in warm water daily. Heating pad use should be avoided. Commercial callus and corn removers should be avoided. The patient should see a specialist to treat these problems.

44. The nurse has administered 4 oz of orange juice to an alert patient whose blood glucose was 62 mg/dL. Fifteen minutes later, the blood glucose is 67 mg/dL. Which action should the nurse take next? a. Give the patient 4 to 6 oz more orange juice. b. Administer the PRN glucagon (Glucagon) 1 mg IM. c. Have the patient eat some peanut butter with crackers. d. Notify the health care provider about the hypoglycemia.

a. Give the patient 4 to 6 oz more orange juice. - The "rule of 15" indicates that administration of quickly acting carbohydrates should be done two or three times for a conscious patient whose glucose remains less than 70 mg/dL before notifying the health care provider. More complex carbohydrates and fats may be used after the glucose has stabilized. Glucagon should be used if the patient's level of consciousness decreases so that oral carbohydrates can no longer be given.

Which are appropriate therapies for patients with diabetes mellitus (select all that apply)? a. Use of statins to reduce CVD risk b. Use of diuretics to treat nephropathy c. Use of ACE inhibitors to treat nephropathy d. Use of serotonin agonists to decrease appetite e. Use of laser photocoagulation to treat retinopathy

a. Use of statins to reduce CVD risk c. Use of ACE inhibitors to treat nephropathy e. Use of laser photocoagulation to treat retinopathy

6. The nurse is assessing a 55-yr-old female patient with type 2 diabetes who has a body mass index (BMI) of 31 kg/m2 .Which goal in the plan of care is most important for this patient? a. The patient will reach a glycosylated hemoglobin level of less than 7%. b. The patient will follow a diet and exercise plan that results in weight loss. c. The patient will choose a diet that distributes calories throughout the day. d. The patient will state the reasons for eliminating simple sugars in the diet.

a. The patient will reach a glycosylated hemoglobin level of less than 7%. - The complications of diabetes are related to elevated blood glucose and the most important patient outcome is the reduction of glucose to near-normal levels. A BMI of 30?9?kg/m2 or above is considered obese, so the other outcomes are appropriate but are not as high in priority.

7. A patient who has type 1 diabetes plans to swim laps for an hour daily at 1:00 PM. The clinic nurse will plan to teach the patient to a. check glucose level before, during, and after swimming. b. delay eating the noon meal until after the swimming class. c. increase the morning dose of neutral protamine Hagedorn (NPH) insulin. d. time the morning insulin injection so that the peak occurs while swimming.

a. check glucose level before, during, and after swimming. - The change in exercise will affect blood glucose, and the patient will need to monitor glucose carefully to determine the need for changes in diet and insulin administration. Because exercise tends to decrease blood glucose, patients are advised to eat before exercising. Increasing the morning NPH or timing the insulin to peak during exercise may lead to hypoglycemia, especially with the increased exercise.

47. After change-of-shift report, which patient should the nurse assess first? a. A 19-yr-old patient with type 1 diabetes who has a hemoglobin A1C of 12% b. A 23-yr-old patient with type 1 diabetes who has a blood glucose of 40 mg/dL c. A 40-yr-old patient who is pregnant and whose oral glucose tolerance test is 202 mg/dL d. A 50-yr-old patient who uses exenatide (Byetta) and is complaining of acute abdominal pain

b. A 23-yr-old patient with type 1 diabetes who has a blood glucose of 40 mg/dL - Because the brain requires glucose to function, untreated hypoglycemia can cause unconsciousness, seizures, and death. The nurse will rapidly assess and treat the patient with low blood glucose. The other patients also have symptoms that require assessments or interventions, but they are not at immediate risk for life-threatening complications.

28. A patient who has diabetes and reported burning foot pain at night receives a new prescription. Which information should the nurse teach the patient about amitriptyline ? a. Amitriptyline decreases the depression caused by your foot pain. b. Amitriptyline helps prevent transmission of pain impulses to the brain. c. Amitriptyline corrects some of the blood vessel changes that cause pain. d. Amitriptyline improves sleep and makes you less aware of nighttime pain.

b. Amitriptyline helps prevent transmission of pain impulses to the brain. - Tricyclic antidepressants (TCAs) decrease the transmission of pain impulses to the spinal cord and brain. TCAs also improve sleep quality and are used for depression, but that is not the major purpose for their use in diabetic neuropathy. The blood vessel changes that contribute to neuropathy are not affected by TCAs.

20. The nurse is preparing to teach a 43-yr-old man who is newly diagnosed with type 2 diabetes about home management of the disease. Which action should the nurse take first? a. Ask the patient's family to participate in the diabetes education program. b. Assess the patient's perception of what it means to have diabetes mellitus. c. Demonstrate how to check glucose using capillary blood glucose monitoring. d. Discuss the need for the patient to actively participate in diabetes management.

b. Assess the patient's perception of what it means to have diabetes mellitus. - Before planning teaching, the nurse should assess the patient's interest in and ability to self-manage the diabetes. After assessing the patient, the other nursing actions may be appropriate, but planning needs to be individualized to each patient.

24. Which action should the nurse take after a patient treated with intramuscular glucagon for hypoglycemia regains consciousness? a. Assess the patient for symptoms of hyperglycemia. b. Give the patient a snack of peanut butter and crackers. c. Have the patient drink a glass of orange juice or nonfat milk. d. Administer a continuous infusion of 5% dextrose for 24 hours.

b. Give the patient a snack of peanut butter and crackers. - Rebound hypoglycemia can occur after glucagon administration, but having a meal containing complex carbohydrates plus protein and fat will help prevent hypoglycemia. Orange juice and nonfat milk will elevate blood glucose rapidly, but the cheese and crackers will stabilize blood glucose. Administration of IV glucose might be used in patients who were unable to take in nutrition orally. The patient should be assessed for symptoms of hypoglycemia after glucagon administration.

15. Which information will the nurse include when teaching a patient who has type 2 diabetes about glyburide ? a. Glyburide decreases glucagon secretion from the pancreas. b. Glyburide stimulates insulin production and release from the pancreas. c. Glyburide should be taken even if the morning blood glucose level is low. d. Glyburide should not be used for 48 hours after receiving IV contrast media.

b. Glyburide stimulates insulin production and release from the pancreas.

38. Which laboratory value reported to the nurse by the unlicensed assistive personnel (UAP) indicates an urgent need for the nurse's assessment of the patient? a. Bedtime glucose of 140 mg/dL b. Noon blood glucose of 52 mg/dL c. Fasting blood glucose of 130 mg/dL d. 2-hr postprandial glucose of 220 mg/dL

b. Noon blood glucose of 52 mg/dL - The nurse should assess the patient with a blood glucose level of 52 mg/dL for symptoms of hypoglycemia and give the patient a carbohydrate-containing beverage such as orange juice. The other values are within an acceptable range or not immediately dangerous for a patient with diabetes

19. The nurse identifies a need for additional teaching when the patient who is self-monitoring blood glucose a. washes the puncture site using warm water and soap. b. chooses a puncture site in the center of the finger pad. c. hangs the arm down for a minute before puncturing the site. d. says the result of 120 mg indicates good blood sugar control.

b. chooses a puncture site in the center of the finger pad. - The patient is taught to choose a puncture site at the side of the finger pad because there are fewer nerve endings along the side of the finger pad. The other patient actions indicate that teaching has been effective.

21. An unresponsive patient with type 2 diabetes is brought to the emergency department and diagnosed with hyperosmolar hyperglycemic syndrome (HHS). The nurse will anticipate the need to a. give 50% dextrose. b. insert an IV catheter. c. initiate O2 by nasal cannula. d. administer glargine (Lantus) insulin.

b. insert an IV catheter. - HHS is initially treated with large volumes of IV fluids to correct hypovolemia. Regular insulin is administered, not a long-acting insulin. There is no indication that the patient requires O2. Dextrose solutions will increase the patient's blood glucose and would be contraindicated.

39. When a patient with type 2 diabetes is admitted for a cholecystectomy, which nursing action can the nurse delegate to a licensed practical/vocational nurse (LPN/LVN)? a. Communicate the blood glucose level and insulin dose to the circulating nurse in surgery. b. Discuss the reason for the use of insulin therapy during the immediate postoperative period. c. Administer the prescribed lispro (Humalog) insulin before transporting the patient to surgery. d. Plan strategies to minimize the risk for hypoglycemia or hyperglycemia during the postoperative period.

c. Administer the prescribed lispro (Humalog) insulin before transporting the patient to surgery. - LPN/LVN education and scope of practice includes administration of insulin. Communication about patient status with other departments, planning, and patient teaching are skills that require RN education and scope of practice.

42. After the nurse has finished teaching a patient who has a new prescription for exenatide (Byetta), which patient statement indicates that the teaching has been effective? a. "I may feel hungrier than usual when I take this medicine." b. "I will not need to worry about hypoglycemia with the Byetta." c. "I should take my daily aspirin at least an hour before the Byetta." d. "I will take the pill at the same time I eat breakfast in the morning."

c. "I should take my daily aspirin at least an hour before the Byetta." - Because exenatide slows gastric emptying, oral medications should be taken at least 1 hour before the exenatide to avoid slowing absorption. Exenatide is injected and increases feelings of satiety. Hypoglycemia can occur with this medication.

1. Which statement by a nurse to a patient newly diagnosed with type 2 diabetes is correct? a. Insulin is not used to control blood glucose in patients with type 2 diabetes. b. Complications of type 2 diabetes are less serious than those of type 1 diabetes. c. Changes in diet and exercise may control blood glucose levels in type 2 diabetes. d. Type 2 diabetes is usually diagnosed when the patient is admitted with a hyperglycemic coma.

c. Changes in diet and exercise may control blood glucose levels in type 2 diabetes.

23. The health care provider suspects the Somogyi effect in a 50-yr-old patient whose 6:00 AM blood glucose is 230 mg/dL. Which action will the nurse teach the patient to take? a. Avoid snacking at bedtime. b. Increase the rapid-acting insulin dose. c. Check the blood glucose during the night d. Administer a larger dose of long-acting insulin.

c. Check the blood glucose during the night - If the Somogyi effect is causing the patient's increased morning glucose level, the patient will experience hypoglycemia between 2:00 and 4:00 AM. The dose of insulin will be reduced, rather than increased. A bedtime snack is used to prevent hypoglycemic episodes during the night.

46. After change-of-shift report, which patient will the nurse assess first? a. A 19-yr-old patient with type 1 diabetes who was admitted with possible dawn phenomenon b. A 35-yr-old patient with type 1 diabetes whose most recent blood glucose reading was 230 mg/dL c. A 60-yr-old patient with hyperosmolar hyperglycemic syndrome who has poor skin turgor and dry oral mucosa d. A 68-yr-old patient with type 2 diabetes who has severe peripheral neuropathy and complains of burning foot pain

c. A 60-yr-old patient with hyperosmolar hyperglycemic syndrome who has poor skin turgor and dry oral mucosa - The patient's diagnosis of HHS and signs of dehydration indicate that the nurse should rapidly assess for signs of shock and determine whether increased fluid infusion is needed. The other patients also need assessment and intervention but do not have life-threatening complications.

43. A few weeks after an 82-yr-old patient with a new diagnosis of type 2 diabetes has been placed on metformin (Glucophage) therapy and taught about appropriate diet and exercise, the home health nurse makes a visit. Which finding should the nurse promptly discuss with the health care provider? a. Hemoglobin A1C level is 7.9%. b. Last eye examination was 18 months ago. c. Glomerular filtration rate is decreased. d. Patient has questions about the prescribed diet.

c. Glomerular filtration rate is decreased. - The decrease in renal function may indicate a need to adjust the dose of metformin or change to a different medication. In older patients, the goal for A1C may be higher in order to avoid complications associated with hypoglycemia. The nurse will plan on scheduling the patient for an eye examination and addressing the questions about diet, but the area for prompt intervention is the patient's decreased renal function.

5. A patient with type 2 diabetes is scheduled for a follow-up visit in the clinic several months from now. Which test will the nurse schedule to evaluate the effectiveness of treatment for the patient? a. Fasting blood glucose b. Oral glucose tolerance c. Glycosylated hemoglobin d. Urine dipstick for glucose

c. Glycosylated hemoglobin - The glycosylated hemoglobin (A1C) test shows the overall control of glucose over 90 to 120 days. A fasting blood level indicates only the glucose level at one time. Urine glucose testing is not an accurate reflection of blood glucose level and does not reflect the glucose over a prolonged time. Oral glucose tolerance testing is done to diagnose diabetes but is not used for monitoring glucose control after diabetes has been diagnosed.

30. Which action by a patient indicates that the home health nurse's teaching about glargine and regular insulin has been successful? a. The patient administers the glargine 30 minutes before each meal. b. The patient's family prefills the syringes with the mix of insulins weekly. c. The patient discards the open vials of glargine and regular insulin after 4 weeks. d. The patient draws up the regular insulin and then the glargine in the same syringe.

c. The patient discards the open vials of glargine and regular insulin after 4 weeks. - Insulin can be stored at room temperature for 4 weeks. Glargine should not be mixed with other insulins or prefilled and stored. Short-acting regular insulin is administered before meals, and glargine is given once daily.

29. A patient who has type 2 diabetes is being prepared for an elective coronary angiogram. Which information would the nurse anticipate might lead to rescheduling the test? a. The patient's most recent A1C was 6.5%. b. The patient's blood glucose is 128 mg/dL. c. The patient took the prescribed metformin today. d. The patient took the prescribed captopril this morning.

c. The patient took the prescribed metformin today. - To avoid lactic acidosis, metformin should be discontinued a day or 2 before the coronary angiogram and should not be used for 48 hours after IV contrast media are administered. The other patient data will also be reported but do not indicate any need to reschedule the procedure.

2. A patient screened for diabetes at a clinic has a fasting plasma glucose level of 120 mg/dL (6.7 mmol/L). The nurse will plan to teach the patient about a. self-monitoring of blood glucose. b. using low doses of regular insulin. c. lifestyle changes to lower blood glucose. d. effects of oral hypoglycemic medications.

c. lifestyle changes to lower blood glucose. - The patient's impaired fasting glucose indicates prediabetes, and the patient should be counseled about lifestyle changes to prevent the development of type 2 diabetes. The patient with prediabetes does not require insulin or oral hypoglycemics for glucose control and does not need to self-monitor blood glucose.

32. The nurse is interviewing a new patient with diabetes who takes rosiglitazone (Avandia). Which information would the nurse anticipate resulting in the health care provider discontinuing the medication? a. The patient's blood pressure is 154/92. b. The patient's blood glucose is 86 mg/dL. c. The patient reports a history of emphysema. d. The patient has chest pressure when walking.

d. The patient has chest pressure when walking. - Rosiglitazone can cause myocardial ischemia. The nurse should immediately notify the health care provider and expect orders to discontinue the medication. A blood glucose level of 86 mg/dL indicates a positive effect from the medication. Hypertension and a history of emphysema do not contraindicate this medication.

3. A 28-yr-old male patient with type 1 diabetes reports how he manages his exercise and glucose control. Which behavior indicates that the nurse should implement additional teaching? a. The patient always carries hard candies when engaging in exercise. b. The patient goes for a vigorous walk when his glucose is 200 mg/dL. c. The patient has a peanut butter sandwich before going for a bicycle ride. d. The patient increases daily exercise when ketones are present in the urine.

d. The patient increases daily exercise when ketones are present in the urine. - When the patient is ketotic, exercise may result in an increase in blood glucose level. Patients with type 1 diabetes should be taught to avoid exercise when ketosis is present. The other statements are correct

Which statement would be correct for a patient with type 2 diabetes who was admitted to the hospital with pneumonia? a. The patient must receive insulin therapy to prevent ketoacidosis. b. The patient has islet cell antibodies that have destroyed the pancreas's ability to produce insulin. c. The patient has minimal or absent endogenous insulin secretion and requires daily insulin injections. d. The patient may have sufficient endogenous insulin to prevent ketosis but is at risk for hyperosmolar hyperglycemic syndrome.

d. The patient may have sufficient endogenous insulin to prevent ketosis but is at risk for hyperosmolar hyperglycemic syndrome.

27. Which finding indicates a need to contact the health care provider before the nurse administers metformin (Glucophage)? a. The patient's blood glucose level is 174 mg/dL. b. The patient is scheduled for a chest x-ray in an hour. c. The patient has gained 2 lb (0.9 kg) in the past 24 hours. d. The patient's blood urea nitrogen (BUN) level is 52 mg/dL

d. The patient's blood urea nitrogen (BUN) level is 52 mg/dL - The BUN indicates possible renal failure, and metformin should not be used in patients with renal failure. The other findings are not contraindications to the use of metformin.

Which nursing action is most important in assisting an older patient who has diabetes to engage in moderate daily exercise? a. Determine what types of activities the patient enjoys. b. Remind the patient that exercise improves self-esteem. c. Teach the patient about the effects of exercise on glucose level. d. Give the patient a list of activities that are moderate in intensity.

A Because consistency with exercise is important, assessment for the types of exercise that the patient finds enjoyable is the most important action by the nurse in ensuring adherence to an exercise program. The other actions may be helpful but are not the most important in improving compliance.

The nurse is preparing to teach a 43-yr-old man who is newly diagnosed with type 2 diabetes about home management of the disease. Which action should the nurse take first? a. Assess the patient's perception of what it means to have diabetes. b. Ask the patient's family to participate in the diabetes education program. c. Demonstrate how to check glucose using capillary blood glucose monitoring. d. Discuss the need for the patient to actively participate in diabetes management.

A Before planning teaching, the nurse should assess the patient's interest in and ability to self-manage the diabetes. After assessing the patient, the other nursing actions may be appropriate, but planning needs to be specific to each patient.

Which nursing action can the nurse delegate to experienced unlicensed assistive personnel (UAP) who are working in the diabetic clinic? a. Measure the ankle-brachial index. b. Check for changes in skin pigmentation. c. Assess for unilateral or bilateral foot drop. d. Ask the patient about symptoms of depression.

A Checking systolic pressure at the ankle and brachial areas and calculating the ankle-brachial index is a procedure that can be done by UAP who have been trained in the procedure. The other assessments require more education and critical thinking and should be done by the registered nurse (RN).

A patient with diabetes is starting on intensive insulin therapy. Which type of insulin will the nurse discuss using for mealtime coverage? a. Lispro (Humalog) b. Glargine (Lantus) c. Detemir (Levemir) d. NPH (Humulin N)

A Rapid- or short-acting insulin is used for mealtime coverage for patients receiving intensive insulin therapy. NPH, glargine, or detemir will be used as the basal insulin.

The nurse is assessing a 55-yr-old female patient with type 2 diabetes who has a body mass index (BMI) of 31 kg/m2 .Which goal in the plan of care is most important for this patient? a. The patient will reach a glycosylated hemoglobin level of less than 7%. b. The patient will follow a diet and exercise plan that results in weight loss. c. The patient will choose a diet that distributes calories throughout the day. d. The patient will state the reasons for eliminating simple sugars in the diet.

A The complications of diabetes are related to elevated blood glucose and the most important patient outcome is the reduction of glucose to near-normal levels. A BMI of 30.9/kg/m2 or above is considered obese, so the other outcomes are appropriate but are not as high in priority.

Which patient with type 1 diabetes mellitus would be at the highest risk for developing hypoglycemic unawareness? A 58-yr-old patient with diabetic retinopathy A 73-yr-old patient who takes propranolol (Inderal) A 19-yr-old patient who is on the school track team A 24-yr-old patient with a hemoglobin A1C of 8.9%

A 73-yr-old patient who takes propranolol (Inderal)

In which order will the nurse take these steps to prepare NPH 20 units and regular insulin 2 units using the same syringe? (Put a comma and a space between each answer choice [A, B, C, D, E]). a. Rotate NPH vial. b. Withdraw regular insulin. c. Withdraw 20 units of NPH. d. Inject 20 units of air into NPH vial. e. Inject 2 units of air into regular insulin vial.

A D E B C

Which patient action indicates an accurate understanding of the nurse's teaching about the use of an insulin pump? a. The patient programs the pump for an insulin bolus after eating. b. The patient changes the location of the insertion site every week. c. The patient takes the pump off at bedtime and starts it again each morning.

A

Which question during the assessment of a patient who has diabetes will help the nurse identify autonomic neuropathy? a. "Do you feel bloated after eating?" b. "Have you seen any skin changes?" c. "Do you need to increase your insulin dosage when you are stressed?" d. "Have you noticed any painful new ulcerations or sores on your feet?"

A

1. In which order will the nurse take these steps to prepare NPH 20 units and regular insulin 2 units using the same syringe? (Put a comma and a space between each answer choice [A, B, C, D, E]). a. Rotate NPH vial. b. Withdraw regular insulin. c. Withdraw 20 units of NPH. d. Inject 20 units of air into NPH vial. e. Inject 2 units of air into regular insulin vial.

A, D, E, B, C

review questions- chapter 48

online

45. Which nursing action can the nurse delegate to experienced unlicensed assistive personnel (UAP) who are working in the diabetic clinic? a. Measure the ankle-brachial index. b. Check for changes in skin pigmentation. c. Assess for unilateral or bilateral foot drop. d. Ask the patient about symptoms of depression.

ANS: A Checking systolic pressure at the ankle and brachial areas and calculating the ankle-brachial index is a procedure that can be done by UAP who have been trained in the procedure. The other assessments require more education and critical thinking and should be done by the registered nurse (RN). DIF: Cognitive Level: Apply (application) REF: 1152 OBJ: Special Questions: Delegation TOP: Nursing Process: Planning MSC: NCLEX: Safe and Effective Care Environment

1. In which order will the nurse take these steps to prepare NPH 20 units and regular insulin 2 units using the same syringe? (Put a comma and a space between each answer choice [A, B, C, D, E]). a. Rotate NPH vial. b. Withdraw regular insulin. c. Withdraw 20 units of NPH. d. Inject 20 units of air into NPH vial. e. Inject 2 units of air into regular insulin vial.

ANS: A,D,E,B,C When mixing regular insulin with NPH, it is important to avoid contact between the regular insulin and the additives in the NPH that slow the onset, peak, and duration of activity in the longer-acting insulin. DIF: Cognitive Level: Analyze (analysis) OBJ: Special Questions: Prioritization TOP: REF: 1126 Nursing Process: Implementation MSC: NCLEX: Physiological Integrity

13. Which patient action indicates a good understanding of the nurse's teaching about the use of an insulin pump? a. The patient programs the pump for an insulin bolus after eating. b. The patient changes the location of the insertion site every week. c. The patient takes the pump off at bedtime and starts it again each morning. d. The patient plans a diet with more calories than usual when using the pump.

ANS: A In addition to the basal rate of insulin infusion, the patient will adjust the pump to administer a bolus after each meal, with the dosage depending on the oral intake. The insertion site should be changed every 2 or 3 days. There is more flexibility in diet and exercise when an insulin pump is used, but it does not provide for consuming a higher calorie diet. The pump will deliver a basal insulin rate 24 hours a day. DIF: Cognitive Level: Apply (application) TOP: Nursing Process: Evaluation REF: 1129 MSC: NCLEX: Health Promotion and Maintenance

44. The nurse has administered 4 oz of orange juice to an alert patient whose blood glucose was 62 mg/dL. Fifteen minutes later, the blood glucose is 67 mg/dL. Which action should the nurse take next? a. Give the patient 4 to 6 oz more orange juice. b. Administer the PRN glucagon (Glucagon) 1 mg IM. c. Have the patient eat some peanut butter with crackers. d. Notify the health care provider about the hypoglycemia.

ANS: A The "rule of 15" indicates that administration of quickly acting carbohydrates should be done two or three times for a conscious patient whose glucose remains less than 70 mg/dL before notifying the health care provider. More complex carbohydrates and fats may be used after the glucose has stabilized. Glucagon should be used if the patient's level of consciousness decreases so that oral carbohydrates can no longer be given. DIF: Cognitive Level: Analyze (analysis) OBJ: Special Questions: Prioritization TOP: REF: 1146 Nursing Process: Implementation MSC: NCLEX: Physiological Integrity

35. A patient with diabetic ketoacidosis is brought to the emergency department. Which prescribed action should the nurse implement first? a. Infuse 1 L of normal saline per hour. b. Give sodium bicarbonate 50 mEq IV push. c. Administer regular insulin 10 U by IV push. d. Start a regular insulin infusion at 0.1 units/kg/hr.

ANS: A The most urgent patient problem is the hypovolemia associated with diabetic ketoacidosis (DKA), and the priority is to infuse IV fluids. The other actions can be done after the infusion of normal saline is initiated. DIF: Cognitive Level: Analyze (analysis) OBJ: Special Questions: Prioritization TOP: REF: 1144 Nursing Process: Implementation MSC: NCLEX: Physiological Integrity

25. Which question during the assessment of a patient who has diabetes will help the nurse identify autonomic neuropathy? a. "Do you feel bloated after eating?" b. "Have you seen any skin changes?" c. "Do you need to increase your insulin dosage when you are stressed?" d. "Have you noticed any painful new ulcerations or sores on your feet?"

ANS: A Autonomic neuropathy can cause delayed gastric emptying, which results in a bloated feeling for the patient. The other questions are also appropriate to ask but would not help in identifying autonomic neuropathy. DIF: Cognitive Level: Apply (application) TOP: Nursing Process: Assessment REF: 1150 MSC: NCLEX: Physiological Integrity

14. A patient with diabetes is starting on intensive insulin therapy. Which type of insulin will the nurse discuss using for mealtime coverage? a. Lispro (Humalog) b. Glargine (Lantus) c. Detemir (Levemir) d. NPH (Humulin N)

ANS: A Rapid- or short- acting insulin is used for mealtime coverage for patients receiving intensive insulin therapy. NPH, glargine, or detemir will be used as the basal insulin. DIF: Cognitive Level: Apply (application) TOP: Nursing Process: Planning REF: 1125 MSC: NCLEX: Physiological Integrity

26. Which information will the nurse include in teaching a female patient who has peripheral arterial disease, type 2 diabetes, and sensory neuropathy of the feet and legs? a. Choose flat-soled leather shoes. b. Set heating pads on a low temperature. c. Use callus remover for corns or calluses. d. Soak feet in warm water for an hour each day.

ANS: A The patient is taught to avoid high heels and that leather shoes are preferred. The feet should be washed, but not soaked, in warm water daily. Heating pad use should be avoided. Commercial callus and corn removers should be avoided. The patient should see a specialist to treat these problems. DIF: Cognitive Level: Apply (application) TOP: Nursing Process: Implementation REF: 1151 MSC: NCLEX: Physiological Integrity

27. Which finding indicates a need to contact the health care provider before the nurse administers metformin (Glucophage)? a. The patient's blood glucose level is 174 mg/dL. b. The patient is scheduled for a chest x-ray in an hour. c. The patient has gained 2 lb (0.9 kg) in the past 24 hours. d. The patient's blood urea nitrogen (BUN) level is 52 mg/dL.

ANS: A The patient is taught to avoid high heels and that leather shoes are preferred. The feet should be washed, but not soaked, in warm water daily. Heating pad use should be avoided. Commercial callus and corn removers should be avoided. The patient should see a specialist to treat these problems. DIF: Cognitive Level: Apply (application) TOP: Nursing Process: Implementation REF: 1151 MSC: NCLEX: Physiological Integrity

12. A patient receives aspart (NovoLog) insulin at 8:00 AM. At which time would the nurse anticipate the highest risk for hypoglycemia? a. 10:00 AM c. 2:00 PM b. 12:00 AM d. 4:0 PM

ANS: A The rapid-acting insulins peak in 1 to 3 hours. The patient is not at a high risk for hypoglycemia at the other listed times, although hypoglycemia may occur. DIF: Cognitive Level: Understand (comprehension) REF: 1132 TOP: Nursing Process: Evaluation MSC: NCLEX: Physiological Integrity

7. A patient who has type 1 diabetes plans to swim laps for an hour daily at 1:00 PM. The clinic nurse will plan to teach the patient to a. check glucose level before, during, and after swimming. b. delay eating the noon meal until after the swimming class. c. increase the morning dose of neutral protamine Hagedorn (NPH) insulin. d. time the morning insulin injection so that the peak occurs while swimming.

ANS: A The change in exercise will affect blood glucose, and the patient will need to monitor glucose carefully to determine the need for changes in diet and insulin administration. Because exercise tends to decrease blood glucose, patients are advised to eat before exercising. Increasing the morning NPH or timing the insulin to peak during exercise may lead to hypoglycemia, especially with the increased exercise. DIF: Cognitive Level: Apply (application) TOP: Nursing Process: Planning REF: 1132 MSC: NCLEX: Physiological Integrity

6. The nurse is assessing a 55-yr-old female patient with type 2 diabetes who has a body mass index (BMI) of 31 kg/m2.Which goal in the plan of care is most important for this patient? a. The patient will reach a glycosylated hemoglobin level of less than 7%. b. The patient will follow a diet and exercise plan that results in weight loss. c. The patient will choose a diet that distributes calories throughout the day. d. The patient will state the reasons for eliminating simple sugars in the diet.

ANS: A The complications of diabetes are related to elevated blood glucose and the most important patient outcome is the reduction of glucose to near-normal levels. A BMI of 30?9?kg/m2 or above is considered obese, so the other outcomes are appropriate but are not as high in priority. DIF: Cognitive Level: Analyze (analysis) REF: 1124 OBJ: Special Questions: Prioritization TOP: Nursing Process: Planning MSC: NCLEX: Physiological Integrity

A patient with diabetic ketoacidosis is brought to the emergency department. Which prescribed action should the nurse implement first? a. Infuse 1 L of normal saline per hour. b. Give sodium bicarbonate 50 mEq IV push. c. Administer regular insulin 10 U by IV push. d. Start a regular insulin infusion at 0.1 units/kg/hr.

ANS: A The most urgent patient problem is the hypovolemia associated with diabetic ketoacidosis (DKA), and the priority is to infuse IV fluids. The other actions can be done after the infusion of normal saline is initiated.

19. The nurse identifies a need for additional teaching when the patient who is self-monitoring blood glucose a. washes the puncture site using warm water and soap. b. chooses a puncture site in the center of the finger pad. c. hangs the arm down for a minute before puncturing the site. d. says the result of 120 mg indicates good blood sugar control.

ANS: B The patient is taught to choose a puncture site at the side of the finger pad because there are fewer nerve endings along the side of the finger pad. The other patient actions indicate that teaching has been effective. DIF: Cognitive Level: Apply (application) TOP: Nursing Process: Evaluation REF: 1136 MSC: NCLEX: Health Promotion and Maintenance

15. Which information will the nurse include when teaching a patient who has type 2 diabetes about glyburide ? a. Glyburide decreases glucagon secretion from the pancreas. b. Glyburide stimulates insulin production and release from the pancreas. c. Glyburide should be taken even if the morning blood glucose level is low. d. Glyburide should not be used for 48 hours after receiving IV contrast media.

ANS: B The sulfonylureas stimulate the production and release of insulin from the pancreas. If the glucose level is low, the patient should contact the health care provider before taking glyburide because hypoglycemia can occur with this class of medication. Metformin should be held for 48 hours after administration of IV contrast media, but this is not necessary for glyburide. Glucagon secretion is not affected by glyburide. DIF: Cognitive Level: Apply (application) TOP: Nursing Process: Implementation REF: 1130 MSC: NCLEX: Physiological Integrity

28. A patient who has diabetes and reported burning foot pain at night receives a new prescription. Which information should the nurse teach the patient about amitriptyline ? a. Amitriptyline decreases the depression caused by your foot pain. b. Amitriptyline helps prevent transmission of pain impulses to the brain. c. Amitriptyline corrects some of the blood vessel changes that cause pain. d. Amitriptyline improves sleep and makes you less aware of nighttime pain.

ANS: B Tricyclic antidepressants (TCAs) decrease the transmission of pain impulses to the spinal cord and brain. TCAs also improve sleep quality and are used for depression, but that is not the major purpose for their use in diabetic neuropathy. The blood vessel changes that contribute to neuropathy are not affected by TCAs. DIF: Cognitive Level: Apply (application) TOP: Nursing Process: Implementation REF: 1150 MSC: NCLEX: Physiological Integrity

11. Which patient action indicates good understanding of the nurse's teaching about administration of aspart (NovoLog) insulin? a. The patient avoids injecting the insulin into the upper abdominal area. b. The patient cleans the skin with soap and water before insulin administration. c. The patient stores the insulin in the freezer after administering the prescribed dose. d. The patient pushes the plunger down while removing the syringe from the injection site.

ANS: B Cleaning the skin with soap and water is acceptable. Insulin should not be frozen. The patient should leave the syringe in place for about 5 seconds after injection to be sure that all the insulin has been injected. The upper abdominal area is one of the preferred areas for insulin injection. DIF: Cognitive Level: Apply (application) TOP: Nursing Process: Evaluation REF: 1128 MSC: NCLEX: Physiological Integrity

21. An unresponsive patient with type 2 diabetes is brought to the emergency department and diagnosed with hyperosmolar hyperglycemic syndrome (HHS). The nurse will anticipate the need to a. give 50% dextrose. b. insert an IV catheter. c. initiate O2 by nasal cannula. d. administer glargine (Lantus) insulin.

ANS: B HHS is initially treated with large volumes of IV fluids to correct hypovolemia. Regular insulin is administered, not a long-acting insulin. There is no indication that the patient requires O2. Dextrose solutions will increase the patient's blood glucose and would be contraindicated. DIF: Cognitive Level: Apply (application) TOP: Nursing Process: Planning REF: 1145 MSC: NCLEX: Physiological Integrity

1. To monitor for complications in a patient with type 2 diabetes, which tests will the nurse in the diabetic clinic schedule at least annually (select all that apply)? a. Chest x-ray b. Blood pressure c. Serum creatinine d. Urine for microalbuminuria e. Complete blood count (CBC) f. Monofilament testing of the foot

ANS: B, C, D, F Blood pressure, serum creatinine, urine testing for microalbuminuria, and monofilament testing of the foot are recommended at least annually to screen for possible microvascular and macrovascular complications of diabetes. Chest x-ray and CBC might be ordered if the patient with diabetes presents with symptoms of respiratory or infectious problems but are not routinely included in screening. DIF: Cognitive Level: Apply (application) TOP: Nursing Process: Planning REF: 1148 MSC: NCLEX: Physiological Integrity

42. After the nurse has finished teaching a patient who has a new prescription for exenatide (Byetta), which patient statement indicates that the teaching has been effective? a. "I may feel hungrier than usual when I take this medicine." b. "I will not need to worry about hypoglycemia with the Byetta." c. "I should take my daily aspirin at least an hour before the Byetta." d. "I will take the pill at the same time I eat breakfast in the morning."

ANS: C Because exenatide slows gastric emptying, oral medications should be taken at least 1 hour before the exenatide to avoid slowing absorption. Exenatide is injected and increases feelings of satiety. Hypoglycemia can occur with this medication. DIF: Cognitive Level: Apply (application) TOP: Nursing Process: Evaluation REF: 1132 MSC: NCLEX: Physiological Integrity

A patient who has type 1 diabetes plans to swim laps for an hour daily at 1:00 PM. What advice should the clinic nurse plan to give the patient? a. Increase the morning dose of NPH insulin (Novolin N). b. Check glucose level before, during, and after swimming. c. Time the morning insulin injection to peak while swimming. d. Delay eating the noon meal until after finishing the swimming.

B The change in exercise will affect blood glucose, and the patient will need to monitor glucose carefully to determine the need for changes in diet and insulin administration. Because exercise tends to decrease blood glucose, patients are advised to eat before exercising. Increasing the morning NPH or timing the insulin to peak during exercise may lead to hypoglycemia, especially with the increased exercise.

A patient with type 2 diabetes is scheduled for a follow-up visit in the clinic several months from now. Which test will the nurse schedule to evaluate the effectiveness of treatment for the patient? a. Fasting blood glucose b. Glycosylated hemoglobin c. Oral glucose tolerance test d. Urine dipstick for glucose and ketones

B The glycosylated hemoglobin (A1C) test shows the overall control of glucose over 90 to 120 days. A fasting blood level indicates only the glucose level at one time. Urine glucose testing is not an accurate reflection of blood glucose level and does not reflect the glucose over a prolonged time. Oral glucose tolerance testing is done to diagnose diabetes but is not used for monitoring glucose control after diabetes has been diagnosed.

The nurse has taught a patient admitted with diabetes principles of foot care. The nurse evaluates that the patient understands the principles of foot care if the patient makes what statement? "I should only walk barefoot in nice dry weather." "I should look at the condition of my feet every day." "I am lucky my shoes fit so nice and tight because they give me firm support." "When I am allowed up out of bed, I should check the shower water with my toes."

"I should look at the condition of my feet every day."

The nurse teaches a patient recently diagnosed with type 1 diabetes mellitus about insulin administration. Which statement by the patient requires an intervention by the nurse? "I will discard any insulin bottle that is cloudy in appearance." "The best injection site for insulin administration is in my abdomen." "I can wash the site with soap and water before insulin administration." "I may keep my insulin at room temperature (75oF) for up to 1 month."

"I will discard any insulin bottle that is cloudy in appearance."

The nurse is teaching a patient with type 2 diabetes mellitus about exercise to help control blood glucose. The nurse knows the patient understands when the patient elicits which exercise plan? "I want to go fishing for 30 minutes each day; I will drink fluids and wear sunscreen." "I will go running each day when my blood sugar is too high to bring it back to normal." "I will plan to keep my job as a teacher because I get a lot of exercise every school day." "I will take a brisk 30-minute walk 5 days per week and do resistance training three times a week."

"I will take a brisk 30-minute walk 5 days per week and do resistance training three times a week."

A patient admitted with type 2 diabetes asks the nurse what "type 2" means. What is the most appropriate response by the nurse? "With type 2 diabetes, the body of the pancreas becomes inflamed." "With type 2 diabetes, the patient is totally dependent on an outside source of insulin." "With type 2 diabetes, insulin secretion is decreased, and insulin resistance is increased." "With type 2 diabetes, the body produces autoantibodies that destroy β-cells in the pancreas."

"With type 2 diabetes, insulin secretion is decreased, and insulin resistance is increased."

A patient is admitted with diabetes mellitus, malnutrition cellulitis, and a potassium level of 5.6 mEq/L. The nurse understands that what could be contributing factors for this laboratory result (select all that apply.)? Select all that apply. 1) The level may be increased as a result of dehydration that accompanies hyperglycemia. 2) The level may be raised as a result of metabolic ketoacidosis caused by hyperglycemia. 3) The level is consistent with renal insufficiency that can develop with renal nephropathy. 4)The patient may be excreting extra sodium and retaining potassium because of malnutrition. 5) This level demonstrates adequate treatment of the cellulitis and effective serum glucose control.

1) The level may be increased as a result of dehydration that accompanies hyperglycemia. 2) The level may be raised as a result of metabolic ketoacidosis caused by hyperglycemia. 3) The level is consistent with renal insufficiency that can develop with renal nephropathy.

A female patient is scheduled for an oral glucose tolerance test. Which information from the patient's health history is important for the nurse to communicate to the health care provider regarding this test? a. The patient uses oral contraceptives. b. The patient runs several days a week. c. The patient has been pregnant three times. d. The patient has a family history of diabetes.

A Oral contraceptive use may falsely elevate oral glucose tolerance test (OGTT) values. Exercise and a family history of diabetes both can affect blood glucose but will not lead to misleading information from the OGTT. History of previous pregnancies may provide informational about gestational glucose tolerance but will not lead to misleading information from the OGTT.

The nurse has administered 4 oz of orange juice to an alert patient whose blood glucose was 62 mg/dL. Fifteen minutes later, the blood glucose is 67 mg/dL. Which action should the nurse take next? a. Give the patient 4 to 6 oz more orange juice. b. Administer the PRN glucagon (Glucagon) 1 mg IM. c. Have the patient eat some peanut butter with crackers. d. Notify the health care provider about the hypoglycemia.

A The "rule of 15" indicates that administration of quickly acting carbohydrates should be done two or three times for a conscious patient whose glucose remains less than 70 mg/dL before notifying the health care provider. More complex carbohydrates and fats may be used after the glucose has stabilized. Glucagon should be used if the patient's level of consciousness decreases so that oral carbohydrates can no longer be given.

34. A 27-yr-old patient admitted with diabetic ketoacidosis (DKA) has a serum glucose level of 732 mg/dL and serum potassium level of 3.1 mEq/L. Which action prescribed by the health care provider should the nurse take first? a. Place the patient on a cardiac monitor. b. Administer IV potassium supplements. c. Ask the patient about home insulin doses. d. Start an insulin infusion at 0.1 units/kg/hr.

ANS: A Hypokalemia can lead to potentially fatal dysrhythmias such as ventricular tachycardia and ventricular fibrillation, which would be detected with electrocardiogram (ECG) monitoring. Because potassium must be infused over at least 1 hour, the nurse should initiate cardiac monitoring before infusion of potassium. Insulin should not be administered without cardiac monitoring because insulin infusion will further decrease potassium levels. Discussion of home insulin and possible causes can wait until the patient is stabilized. DIF: Cognitive Level: Analyze (analysis) REF: 1146 OBJ: Special Questions: Prioritization TOP: Nursing Process: Implementation MSC: NCLEX: Physiological Integrity

37. A female patient is scheduled for an oral glucose tolerance test. Which information from the patient's health history is important for the nurse to communicate to the health care provider regarding this test? a. The patient uses oral contraceptives. b. The patient runs several days a week. c. The patient has been pregnant three times. d. The patient has a family history of diabetes.

ANS: A Oral contraceptive use may falsely elevate oral glucose tolerance test (OGTT) values. Exercise and a family history of diabetes both can affect blood glucose but will not lead to misleading information from the OGTT. History of previous pregnancies may provide informational about gestational glucose tolerance but will not lead to misleading information from the OGTT. DIF: Cognitive Level: Apply (application) TOP: Nursing Process: Assessment REF: 1124 MSC: NCLEX: Physiological Integrity

41. A 30-yr-old patient has a new diagnosis of type 2 diabetes. The nurse will discuss the need to schedule a dilated eye examination a. every 2 years. b. as soon as possible. c. when the patient is 39 years old. d. within the first year after diagnosis.

ANS: B Because many patients have some diabetic retinopathy when they are first diagnosed with type 2 diabetes, a dilated eye examination is recommended at the time of diagnosis and annually thereafter. Patients with type 1 diabetes should have dilated eye examinations starting 5 years after they are diagnosed and then annually. DIF: Cognitive Level: Apply (application) TOP: Nursing Process: Planning REF: 1149 MSC: NCLEX: Physiological Integrity

38. Which laboratory value reported to the nurse by the unlicensed assistive personnel (UAP) indicates an urgent need for the nurse's assessment of the patient? a. Bedtime glucose of 140 mg/dL b. Noon blood glucose of 52 mg/dL c. Fasting blood glucose of 130 mg/dL d. 2-hr postprandial glucose of 220 mg/dL

ANS: B The nurse should assess the patient with a blood glucose level of 52 mg/dL for symptoms of hypoglycemia and give the patient a carbohydrate- containing beverage such as orange juice. The other values are within an acceptable range or not immediately dangerous for a patient with diabetes. DIF: Cognitive Level: Apply (application) TOP: Nursing Process: Assessment REF: 1152 MSC: NCLEX: Physiological Integrity

47. After change-of-shift report, which patient should the nurse assess first? a. A 19-yr-old patient with type 1 diabetes who has a hemoglobin A1C of 12% b. A 23-yr-old patient with type 1 diabetes who has a blood glucose of 40 mg/dL c. A 40-yr-old patient who is pregnant and whose oral glucose tolerance test is 202 mg/dL d. A 50-yr-old patient who uses exenatide (Byetta) and is complaining of acute abdominal pain

ANS: B Because the brain requires glucose to function, untreated hypoglycemia can cause unconsciousness, seizures, and death. The nurse will rapidly assess and treat the patient with low blood glucose. The other patients also have symptoms that require assessments or interventions, but they are not at immediate risk for life-threatening complications. DIF: Cognitive Level: Analyze (analysis) REF: 1146 OBJ: Special Questions: Prioritization | Special Questions: Multiple Patients TOP: Nursing Process: Planning MSC: NCLEX: Safe and Effective Care Environment

20. The nurse is preparing to teach a 43-yr-old man who is newly diagnosed with type 2 diabetes about home management of the disease. Which action should the nurse take first? a. Ask the patient's family to participate in the diabetes education program. b. Assess the patient's perception of what it means to have diabetes mellitus. c. Demonstrate how to check glucose using capillary blood glucose monitoring. d. Discuss the need for the patient to actively participate in diabetes management.

ANS: B Before planning teaching, the nurse should assess the patient's interest in and ability to self-manage the diabetes. After assessing the patient, the other nursing actions may be appropriate, but planning needs to be individualized to each patient. DIF: Cognitive Level: Analyze (analysis) REF: 1139 OBJ: Special Questions: Prioritization TOP: Nursing Process: Planning MSC: NCLEX: Health Promotion and Maintenance

8. The nurse determines a need for additional instruction when the patient with newly diagnosed type 1 diabetes says which of the following? a. "I will need a bedtime snack because I take an evening dose of NPH insulin." b. "I can choose any foods, as long as I use enough insulin to cover the calories." c. "I can have an occasional beverage with alcohol if I include it in my meal plan." d. "I will eat something at meal times to prevent hypoglycemia, even if I am not hungry."

ANS: B Most patients with type 1 diabetes need to plan diet choices very carefully. Patients who are using intensified insulin therapy have considerable flexibility in diet choices but still should restrict dietary intake of items such as fat, protein, and alcohol. The other patient statements are correct and indicate good understanding of the diet instruction. DIF: Cognitive Level: Apply (application) TOP: Nursing Process: Evaluation REF: 1132 MSC: NCLEX: Physiological Integrity

33. The nurse is taking a health history from a 29-yr-old pregnant patient at the first prenatal visit. The patient reports that she has no personal history of diabetes, but her mother has diabetes. Which action will the nurse plan to take? a. Teach the patient about administering regular insulin. b. Schedule the patient for a fasting blood glucose level. c. Teach about an increased risk for fetal problems with gestational diabetes. d. Schedule an oral glucose tolerance test for the twenty-fourth week of pregnancy.

ANS: B Patients at high risk for gestational diabetes should be screened for diabetes on the initial prenatal visit. An oral glucose tolerance test may also be used to check for diabetes, but it would be done before the twenty-fourth week. Teaching plans would depend on the outcome of a fasting blood glucose test and other tests. DIF: Cognitive Level: Apply (application) REF: 1138 OBJ: Special Questions: Prioritization TOP: Nursing Process: Planning MSC: NCLEX: Physiological Integrity

40. An active 32-yr-old male who has type 1 diabetes is being seen in the endocrine clinic. Which finding indicates a need for the nurse to discuss a possible a change in therapy with the health care provider? a. Hemoglobin A1C level of 6.2% b. Blood pressure of 140/88 mmHg c. Heart rate at rest of 58 beats/minute d. High density lipoprotein (HDL) level of 65 mg/dL

ANS: B To decrease the incidence of macrovascular and microvascular problems in patients with diabetes, the goal blood pressure is usually 130/80 mm Hg. An A1C less than 6.5%, a low resting heart rate (consistent with regular aerobic exercise in a young adult), and an HDL level of 65 mg/dL all indicate that the patient's diabetes and risk factors for vascular disease are well controlled. DIF: Cognitive Level: Apply (application) TOP: Nursing Process: Assessment REF: 1148 MSC: NCLEX: Physiological Integrity

17. When a patient who takes metformin (Glucophage) to manage type 2 diabetes develops an allergic rash from an unknown cause, the health care provider prescribes prednisone. The nurse will anticipate that the patient may a. need a diet higher in calories while receiving prednisone. b. develop acute hypoglycemia while taking the prednisone. c. require administration of insulin while taking prednisone. d. have rashes caused by metformin-prednisone interactions.

ANS: C Glucose levels increase when patients are taking corticosteroids, and insulin may be required to control blood glucose. Hypoglycemia is not a side effect of prednisone. Rashes are not an adverse effect caused by taking metformin and prednisone simultaneously. The patient may have an increased appetite when taking prednisone but will not need a diet that is higher in calories. DIF: Cognitive Level: Apply (application) TOP: Nursing Process: Planning REF: 1124 MSC: NCLEX: Physiological Integrity

22. A 26-yr-old female with type 1 diabetes develops a sore throat and runny nose after caring for her sick toddler. The patient calls the clinic for advice about her symptoms and a blood glucose level of 210 mg/dL despite taking her usual glargine (Lantus) and lispro (Humalog) insulin. The nurse advises the patient to a. use only the lispro insulin until the symptoms are resolved. b. limit intake of calories until the glucose is less than 120 mg/dL. c. monitor blood glucose every 4 hours and notify the clinic if it continues to rise. d. decrease intake of carbohydrates until glycosylated hemoglobin is less than 7%.

ANS: C Infection and other stressors increase blood glucose levels and the patient will need to test blood glucose frequently, treat elevations appropriately with lispro insulin, and call the health care provider if glucose levels continue to be elevated. Discontinuing the glargine will contribute to hyperglycemia and may lead to diabetic ketoacidosis (DKA). Decreasing carbohydrate or caloric intake is not appropriate because the patient will need more calories when ill. Glycosylated hemoglobin testing is not used to evaluate short-term alterations in blood glucose. DIF: Cognitive Level: Apply (application) TOP: Nursing Process: Implementation REF: 1139 MSC: NCLEX: Physiological Integrity

30. Which action by a patient indicates that the home health nurse's teaching about glargine and regular insulin has been successful? a. The patient administers the glargine 30 minutes before each meal. b. The patient's family prefills the syringes with the mix of insulins weekly. c. The patient discards the open vials of glargine and regular insulin after 4 weeks. d. The patient draws up the regular insulin and then the glargine in the same syringe.

ANS: C Insulin can be stored at room temperature for 4 weeks. Glargine should not be mixed with other insulins or prefilled and stored. Short-acting regular insulin is administered before meals, and glargine is given once daily. DIF: Cognitive Level: Apply (application) TOP: Nursing Process: Evaluation REF: 1127 MSC: NCLEX: Physiological Integrit

43. A few weeks after an 82-yr-old patient with a new diagnosis of type 2 diabetes has been placed on metformin (Glucophage) therapy and taught about appropriate diet and exercise, the home health nurse makes a visit. Which finding should the nurse promptly discuss with the health care provider? a. Hemoglobin A1C level is 7.9%. b. Last eye examination was 18 months ago. c. Glomerular filtration rate is decreased. d. Patient has questions about the prescribed diet.

ANS: C The decrease in renal function may indicate a need to adjust the dose of metformin or change to a different medication. In older patients, the goal for A1C may be higher in order to avoid complications associated with hypoglycemia. The nurse will plan on scheduling the patient for an eye examination and addressing the questions about diet, but the area for prompt intervention is the patient's decreased renal function. DIF: Cognitive Level: Apply (application) TOP: Nursing Process: Assessment REF: 1130 MSC: NCLEX: Physiological Integrity

36. A patient who was admitted with diabetic ketoacidosis secondary to a urinary tract infection has been weaned off an insulin drip 30 minutes ago. The patient reports feeling lightheaded and sweaty. Which action should the nurse take first? a. Infuse dextrose 50% by slow IV push. b. Administer 1 mg glucagon subcutaneously. c. Obtain a glucose reading using a finger stick. d. Have the patient drink 4 ounces of orange juice.

ANS: C The patient's clinical manifestations are consistent with hypoglycemia, and the initial action should be to check the patient's glucose with a finger stick or order a stat blood glucose. If the glucose is low, the patient should ingest a rapid-acting carbohydrate, such as orange juice. Glucagon or dextrose 50% might be given if the patient's symptoms become worse or if the patient is unconscious. DIF: Cognitive Level: Analyze (analysis) OBJ: Special Questions: Prioritization TOP: REF: 1135 Nursing Process: Implementation MSC: NCLEX: Physiological Integrity

46. After change-of-shift report, which patient will the nurse assess first? a. A 19-yr-old patient with type 1 diabetes who was admitted with possible dawn phenomenon b. A 35-yr-old patient with type 1 diabetes whose most recent blood glucose reading was 230 mg/dL c. A 60-yr-old patient with hyperosmolar hyperglycemic syndrome who has poor skin turgor and dry oral mucosa d. A 68-yr-old patient with type 2 diabetes who has severe peripheral neuropathy and complains of burning foot pain

ANS: C The patient's diagnosis of HHS and signs of dehydration indicate that the nurse should rapidly assess for signs of shock and determine whether increased fluid infusion is needed. The other patients also need assessment and intervention but do not have life-threatening complications. DIF: Cognitive Level: Analyze (analysis) REF: 1146 OBJ: Special Questions: Multiple Patients | Special Questions: Prioritization TOP: Nursing Process: Planning MSC: NCLEX: Safe and Effective Care Environment

18. A hospitalized diabetic patient received 38 U of NPH insulin at 7:00 AM. At 1:00 PM, the patient has been away from the nursing unit for 2 hours, missing the lunch delivery while awaiting a chest x-ray. To prevent hypoglycemia, the best action by the nurse is to a. save the lunch tray for the patient's later return to the unit. b. ask that diagnostic testing area staff to start a 5% dextrose IV. c. send a glass of milk or orange juice to the patient in the diagnostic testing area. d. request that if testing is further delayed, the patient be returned to the unit to eat.

ANS: D Consistency for mealtimes assists with regulation of blood glucose, so the best option is for the patient to have lunch at the usual time. Waiting to eat until after the procedure is likely to cause hypoglycemia. Administration of an IV solution is unnecessarily invasive for the patient. A glass of milk or juice will keep the patient from becoming hypoglycemic but will cause a rapid rise in blood glucose because of the rapid absorption of the simple carbohydrate in these items. DIF: Cognitive Level: Analyze (analysis) TOP: Nursing Process: Implementation MSC: REF: 1127 NCLEX: Physiological Integrity

A patient who has diabetes and reports burning foot pain at night receives a new prescription. Which information should the nurse teach the patient about amitriptyline? a. Amitriptyline decreases the depression caused by your foot pain. b. Amitriptyline helps prevent transmission of pain impulses to the brain. c. Amitriptyline corrects some of the blood vessel changes that cause pain. d. Amitriptyline improves sleep and makes you less aware of nighttime pain.

B Tricyclic antidepressants (TCAs) decrease the transmission of pain impulses to the spinal cord and brain. TCAs also improve sleep quality and are used for depression, but that is not the major purpose for their use in diabetic neuropathy. TCAs do not affect the blood vessel changes that contribute to neuropathy.

The patient with type 1 diabetes mellitus eats a large meal but does not take his insulin as prescribed. In what order do the following physiologic events occur in the development of a hyperglycemic emergency? (Answer with a letter followed by a comma and a space (e.g. A, B, C, D).) a. Fat metabolism leads to ketonemia. b. Insufficient insulin in the blood stream c. Fat is mobilized for energy from the adipose tissue .d. Potassium and water are excreted with H+ and ketones in the urine. e. Organic acid accumulation in the blood causes metabolic acidosis.

B, C, A, E, D

To monitor for complications in a patient with type 2 diabetes, which tests will the nurse in the diabetic clinic schedule at least annually? (Select all that apply.) a. Chest x-ray b. Blood pressure c. Serum creatinine d. Urine for microalbuminuria e. Complete blood count (CBC) f. Monofilament testing of the foot

BCDF

A hospitalized patient who is diabetic received 38 U of NPH insulin at 7:00 AM. At 1:00 PM, the patient has been away from the nursing unit for 2 hours, missing the lunch delivery while awaiting a chest x-ray. What is the best action by the nurse to prevent hypoglycemia? a. Plan to discontinue the evening dose of insulin. b. Save the lunch tray for the patient's later return. c. Request that if testing is further delayed, the patient will eat lunch first. d. Send a glass of orange juice to the patient in the diagnostic testing area.

C Consistency for mealtimes assists with regulation of blood glucose, so the best option is for the patient to have lunch at the usual time. Waiting to eat until after the procedure is likely to cause hypoglycemia. Holding the insulin dose later will not prevent hypoglycemia form the peak of the NPH dose. A glass of juice will keep the patient from becoming hypoglycemic but will cause a rapid rise in blood glucose because of the rapid absorption of the simple carbohydrate in these items.

Which statement by a nurse to a patient newly diagnosed with type 2 diabetes is accurate? a. Insulin is not used to control blood glucose in patients with type 2 diabetes. b. Complications of type 2 diabetes are less serious than those of type 1 diabetes. c. Changes in diet and exercise may control blood glucose levels in type 2 diabetes. d. Type 2 diabetes is usually diagnosed when a patient is admitted in hyperglycemic coma.

C For some patients with type 2 diabetes, changes in lifestyle are sufficient to achieve blood glucose control. Insulin is frequently used for type 2 diabetes, complications are equally severe as for type 1 diabetes, and type 2 diabetes is usually diagnosed with routine laboratory testing or after a patient develops complications such as frequent yeast infections.

When a patient who takes metformin (Glucophage) to manage type 2 diabetes develops an allergic rash from an unknown cause, the health care provider prescribes prednisone. What should the nurse anticipate? a. The patient may need a diet higher in calories while receiving prednisone. b. The patient may develop acute hypoglycemia while taking the prednisone. c. The patient may require administration of insulin while taking prednisone. d. The patient may have rashes caused by metformin-prednisone interactions.

C Glucose levels increase when patients are taking corticosteroids, and insulin may be required to control blood glucose. Hypoglycemia is not a side effect of prednisone. Rashes are not an adverse effect caused by taking metformin and prednisone simultaneously. The patient may have an increased appetite when taking prednisone but will not need a diet that is higher in calories.

Which action by a patient indicates that the home health nurse's teaching about glargine and regular insulin has been successful? a. The patient administers the glargine 30 minutes before each meal. b. The patient's family prefills the syringes with the mix of insulins weekly. c. The patient discards the open vials of glargine and regular insulin after 4 weeks. d. The patient draws up the regular insulin and then the glargine in the same syringe.

C Insulin can be stored at room temperature for 4 weeks. Glargine should not be mixed with other insulins or prefilled and stored. Short-acting regular insulin is administered before meals, and glargine is given once daily.

When a patient with type 2 diabetes is admitted for a cholecystectomy, which nursing action can the nurse delegate to a licensed practical/vocational nurse (LPN/VN)? a. Communicate the blood glucose level and insulin dose to the circulating nurse in surgery. b. Discuss the reason for the use of insulin therapy during the immediate postoperative period. c. Administer the prescribed lispro (Humalog) insulin before transporting the patient to surgery. d. Plan strategies to minimize the risk for hypoglycemia or hyperglycemia during the postoperative period.

C LPN/LVN education and scope of practice includes administration of insulin. Communication about patient status with other departments, planning, and patient teaching are skills that require RN education and scope of practice.

A patient with diabetes rides a bicycle to and from work every day. Which site should the nurse teach the patient to use to administer the morning insulin? a. Thigh b. Buttock c. Abdomen d. Upper arm

C Patients should be taught not to administer insulin into a site that will be exercised because exercise will increase the rate of absorption. The thigh, buttock, and arm are all exercised by riding a bicycle.

A patient who was admitted with diabetic ketoacidosis secondary to a urinary tract infection has been weaned off an insulin drip 30 minutes ago. The patient reports feeling lightheaded and sweaty. Which action should the nurse take first? a. Infuse dextrose 50% by slow IV push. b. Administer 1 mg glucagon subcutaneously. c. Obtain a glucose reading using a finger stick. d. Have the patient drink 4 ounces of orange juice.

C The patient's clinical manifestations are consistent with hypoglycemia, and the initial action should be to check the patient's glucose with a finger stick or order a stat blood glucose. If the glucose is low, the patient should ingest a rapid-acting carbohydrate, such as orange juice. Glucagon or dextrose 50% might be given if the patient's symptoms become worse or if the patient is unconscious.

A patient screened for diabetes at a clinic has a fasting plasma glucose level of 120 mg/dL (6.7 mmol/L). What should the nurse plan to teach the patient? a. Self-monitoring of blood glucose b. Using low doses of regular insulin c. Lifestyle changes to lower blood glucose d. Effects of oral hypoglycemic medications

C The patient's impaired fasting glucose indicates prediabetes, and the patient should be counseled about lifestyle changes to prevent the development of type 2 diabetes. The patient with prediabetes does not require insulin or oral hypoglycemics for glucose control and does not need to self-monitor blood glucose.

The nurse is assessing a 22-yr-old patient experiencing the onset of symptoms of type 1 diabetes. To which question would the nurse anticipate a positive response? a. "Are you anorexic?" b. "Is your urine dark colored?" c. "Have you lost weight lately?" d. "Do you crave sugary drinks?"

C Weight loss occurs because the body is no longer able to absorb glucose and starts to break down protein and fat for energy. The patient is thirsty but does not necessarily crave sugar-containing fluids. Increased appetite is a classic symptom of type 1 diabetes. With the classic symptom of polyuria, urine will be very dilute.

A 28-yr-old male patient with type 1 diabetes reports how he manages his exercise and glucose control. Which behavior indicates that the nurse should implement additional teaching? a. The patient always carries hard candies when engaging in exercise. b. The patient goes for a vigorous walk when his glucose is 200 mg/dL. c. The patient has a peanut butter sandwich before going for a bicycle ride. d. The patient increases daily exercise when ketones are present in the urine.

D When the patient is ketotic, exercise may result in an increase in blood glucose level. Patients with type 1 diabetes should be taught to avoid exercise when ketosis is present. The other statements are correct.

The nurse caring for a patient hospitalized with diabetes mellitus would look for which laboratory test result to obtain information on the patient's past glucose control? Prealbumin level Urine ketone level Fasting glucose level Glycosylated hemoglobin level

Glycosylated hemoglobin level

a 27yr old admitted with DKA has a serum glucose level of 732 mg/dL and serum potassium level of 3.1 mEq/L. Which action prescribed by the HCP should the nurse take first?

Place pt on cardiac monitor.

Which statement by the patient with type 2 diabetes is accurate? a. "I will limit my alcohol intake to one drink." b. "I am not allowed to eat any sweets because of my diabetes." c. "I cannot exercise because I take a blood glucose-lowering medication." d. "The amount of fat in my diet is not important. Only carbohydrates raise my blood sugar."

a. "I will limit my alcohol intake to one drink."

9. To assist an older patient with diabetes to engage in moderate daily exercise, which action is most important for the nurse to take? a. Determine what types of activities the patient enjoys. b. Remind the patient that exercise improves self-esteem. c. Teach the patient about the effects of exercise on glucose level. d. Give the patient a list of activities that are moderate in intensity

a. Determine what types of activities the patient enjoys.

35. A patient with diabetic ketoacidosis is brought to the emergency department. Which prescribed action should the nurse implement first? a. Infuse 1 L of normal saline per hour. b. Give sodium bicarbonate 50 mEq IV push. c. Administer regular insulin 10 U by IV push. d. Start a regular insulin infusion at 0.1 units/kg/hr.

a. Infuse 1 L of normal saline per hour. - The most urgent patient problem is the hypovolemia associated with diabetic ketoacidosis (DKA), and the priority is to infuse IV fluids. The other actions can be done after the infusion of normal saline is initiated.

You are caring for a patient with newly diagnosed type 1 diabetes. What information is essential to include in your patient teaching before discharge from the hospital (select all that apply)? a. Insulin administration b. Elimination of sugar from diet c. Need to reduce physical activity d. Use of a portable blood glucose monitor e. Hypoglycemia prevention, symptoms, and treatment

a. Insulin administration d. Use of a portable blood glucose monitor e. Hypoglycemia prevention, symptoms, and treatment

45. Which nursing action can the nurse delegate to experienced unlicensed assistive personnel (UAP) who are working in the diabetic clinic? a. Measure the ankle-brachial index. b. Check for changes in skin pigmentation. c. Assess for unilateral or bilateral foot drop. d. Ask the patient about symptoms of depression.

a. Measure the ankle-brachial index. - Checking systolic pressure at the ankle and brachial areas and calculating the ankle-brachial index is a procedure that can be done by UAP who have been trained in the procedure. The other assessments require more education and critical thinking and should be done by the registered nurse (RN).

34. A 27-yr-old patient admitted with diabetic ketoacidosis (DKA) has a serum glucose level of 732 mg/dL and serum potassium level of 3.1 mEq/L. Which action prescribed by the health care provider should the nurse take first? a. Place the patient on a cardiac monitor. b. Administer IV potassium supplements. c. Ask the patient about home insulin doses. d. Start an insulin infusion at 0.1 units/kg/hr.

a. Place the patient on a cardiac monitor. - Hypokalemia can lead to potentially fatal dysrhythmias such as ventricular tachycardia and ventricular fibrillation, which would be detected with electrocardiogram (ECG) monitoring. Because potassium must be infused over at least 1 hour, the nurse should initiate cardiac monitoring before infusion of potassium. Insulin should not be administered without cardiac monitoring because insulin infusion will further decrease potassium levels. Discussion of home insulin and possible causes can wait until the patient is stabilized.

13. Which patient action indicates a good understanding of the nurse's teaching about the use of an insulin pump? a. The patient programs the pump for an insulin bolus after eating. b. The patient changes the location of the insertion site every week. c. The patient takes the pump off at bedtime and starts it again each morning. d. The patient plans a diet with more calories than usual when using the pump.

a. The patient programs the pump for an insulin bolus after eating. - In addition to the basal rate of insulin infusion, the patient will adjust the pump to administer a bolus after each meal, with the dosage depending on the oral intake. The insertion site should be changed every 2 or 3 days. There is more flexibility in diet and exercise when an insulin pump is used, but it does not provide for consuming a higher calorie diet. The pump will deliver a basal insulin rate 24 hours a day.

37. A female patient is scheduled for an oral glucose tolerance test. Which information from the patient's health history is important for the nurse to communicate to the health care provider regarding this test? a. The patient uses oral contraceptives. b. The patient runs several days a week. c. The patient has been pregnant three times. d. The patient has a family history of diabetes.

a. The patient uses oral contraceptives. - Oral contraceptive use may falsely elevate oral glucose tolerance test (OGTT) values. Exercise and a family history of diabetes both can affect blood glucose but will not lead to misleading information from the OGTT. History of previous pregnancies may provide informational about gestational glucose tolerance but will not lead to misleading information from the OGTT.

8. The nurse determines a need for additional instruction when the patient with newly diagnosed type 1 diabetes says which of the following? a. "I will need a bedtime snack because I take an evening dose of NPH insulin." b. "I can choose any foods, as long as I use enough insulin to cover the calories." c. "I can have an occasional beverage with alcohol if I include it in my meal plan." d. "I will eat something at meal times to prevent hypoglycemia, even if I am not hungry."

b. "I can choose any foods, as long as I use enough insulin to cover the calories." - Most patients with type 1 diabetes need to plan diet choices very carefully. Patients who are using intensified insulin therapy have considerable flexibility in diet choices but still should restrict dietary intake of items such as fat, protein, and alcohol. The other patient statements are correct and indicate good understanding of the diet instruction.

33. The nurse is taking a health history from a 29-yr-old pregnant patient at the first prenatal visit. The patient reports that she has no personal history of diabetes, but her mother has diabetes. Which action will the nurse plan to take? a. Teach the patient about administering regular insulin. b. Schedule the patient for a fasting blood glucose level. c. Teach about an increased risk for fetal problems with gestational diabetes. d. Schedule an oral glucose tolerance test for the twenty-fourth week of pregnancy.

b. Schedule the patient for a fasting blood glucose level.

11. Which patient action indicates good understanding of the nurse's teaching about administration of aspart (NovoLog) insulin? a. The patient avoids injecting the insulin into the upper abdominal area. b. The patient cleans the skin with soap and water before insulin administration. c. The patient stores the insulin in the freezer after administering the prescribed dose. d. The patient pushes the plunger down while removing the syringe from the injection site.

b. The patient cleans the skin with soap and water before insulin administration. - Cleaning the skin with soap and water is acceptable. Insulin should not be frozen. The patient should leave the syringe in place for about 5 seconds after injection to be sure that all the insulin has been injected. The upper abdominal area is one of the preferred areas for insulin injection.

41. A 30-yr-old patient has a new diagnosis of type 2 diabetes. The nurse will discuss the need to schedule a dilated eye examination a. every 2 years. b. as soon as possible. c. when the patient is 39 years old. d. within the first year after diagnosis.

b. as soon as possible. - Because many patients have some diabetic retinopathy when they are first diagnosed with type 2 diabetes, a dilated eye examination is recommended at the time of diagnosis and annually thereafter. Patients with type 1 diabetes should have dilated eye examinations starting 5 years after they are diagnosed and then annually.

Polydipsia and polyuria related to diabetes mellitus are primarily due to a. the release of ketones from cells during fat metabolism. b. fluid shifts resulting from the osmotic effect of hyperglycemia. c. damage to the kidneys from exposure to high levels of glucose. d. changes in RBCs resulting from attachment of excessive glucose to hemoglobin.

b. fluid shifts resulting from the osmotic effect of hyperglycemia.

36. A patient who was admitted with diabetic ketoacidosis secondary to a urinary tract infection has been weaned off an insulin drip 30 minutes ago. The patient reports feeling lightheaded and sweaty. Which action should the nurse take first? a. Infuse dextrose 50% by slow IV push. b. Administer 1 mg glucagon subcutaneously. c. Obtain a glucose reading using a finger stick. d. Have the patient drink 4 ounces of orange juice.

c. Obtain a glucose reading using a finger stick. - The patient's clinical manifestations are consistent with hypoglycemia, and the initial action should be to check the patient's glucose with a finger stick or order a stat blood glucose. If the glucose is low, the patient should ingest a rapid-acting carbohydrate, such as orange juice. Glucagon or dextrose 50% might be given if the patient's symptoms become worse or if the patient is unconscious.

31. A patient with diabetes rides a bicycle to and from work every day. Which site should the nurse teach the patient to use to administer the morning insulin? a. thigh. c. abdomen. b. buttock. d. upper arm.

c. abdomen.

22. A 26-yr-old female with type 1 diabetes develops a sore throat and runny nose after caring for her sick toddler. The patient calls the clinic for advice about her symptoms and a blood glucose level of 210 mg/dL despite taking her usual glargine (Lantus) and lispro (Humalog) insulin. The nurse advises the patient to a. use only the lispro insulin until the symptoms are resolved. b. limit intake of calories until the glucose is less than 120 mg/dL. c. monitor blood glucose every 4 hours and notify the clinic if it continues to rise. d. decrease intake of carbohydrates until glycosylated hemoglobin is less than 7%.

c. monitor blood glucose every 4 hours and notify the clinic if it continues to rise. - Infection and other stressors increase blood glucose levels and the patient will need to test blood glucose frequently, treat elevations appropriately with lispro insulin, and call the health care provider if glucose levels continue to be elevated. Discontinuing the glargine will contribute to hyperglycemia and may lead to diabetic ketoacidosis (DKA). Decreasing carbohydrate or caloric intake is not appropriate because the patient will need more calories when ill. Glycosylated hemoglobin testing is not used to evaluate short-term alterations in blood glucose.

A patient with diabetes has a serum glucose level of 824 mg/dL (45.7 mmol/L) and is unresponsive. After assessing the patient, the nurse suspects diabetic ketoacidosis rather than hyperosmolar hyperglycemic syndrome based on the finding of a. polyuria. b. severe dehydration. c. rapid, deep respirations. d. decreased serum potassium.

c. rapid, deep respirations.

17. When a patient who takes metformin (Glucophage) to manage type 2 diabetes develops an allergic rash from an unknown cause, the health care provider prescribes prednisone. The nurse will anticipate that the patient may a. need a diet higher in calories while receiving prednisone. b. develop acute hypoglycemia while taking the prednisone. c. require administration of insulin while taking prednisone. d. have rashes caused by metformin-prednisone interactions.

c. require administration of insulin while taking prednisone. - Glucose levels increase when patients are taking corticosteroids, and insulin may be required to control blood glucose. Hypoglycemia is not a side effect of prednisone. Rashes are not an adverse effect caused by taking metformin and prednisone simultaneously. The patient may have an increased appetite when taking prednisone but will not need a diet that is higher in calories.

16. The nurse has been teaching a patient with type 2 diabetes about managing blood glucose levels and taking glipizide (Glucotrol). Which patient statement indicates a need for additional teaching? a. "If I overeat at a meal, I will still take the usual dose of medication." b. "Other medications besides the Glucotrol may affect my blood sugar." c. "When I am ill, I may have to take insulin to control my blood sugar." d. "My diabetes won't cause complications because I don't need insulin."

d. "My diabetes won't cause complications because I don't need insulin." - The patient should understand that type 2 diabetes places the patient at risk for many complications and that good glucose control is as important when taking oral agents as when using insulin. The other statements are accurate and indicate good understanding of the use of glipizide.


Kaugnay na mga set ng pag-aaral

Chapter 16: Gene regulation in bacteria

View Set

Promulgated Contract Forms Practice Test

View Set

Wk 4 - Practice: Ch. 13, Building the Price Foundation [due Day 5]

View Set

Teorie biofyzika- STAVBA HMOTY, MOLEKULOVÁ FYZIKA

View Set

CCNA Cybersecurity Operations (Version 1.1) - CyberOps Chapter 13 Exam

View Set

Chapter 16: Working with the Command-Line Interface

View Set

Anthropology 204- Language and Culture Exam

View Set